subject
Mathematics, 22.04.2020 01:36 anthonysf

Using strong induction (so, neither weak nor "weak++") on the number of matches in the pile, prove that if this number is of the form 4k + 1, for k ∈ N, then P2 always has a strategy to win (which they will follow, because they are smart). Otherwise, P1 always have a strategy to win (which they will also follow because they’re smart too)! To be clear, you will need to prove both facts with a single inductive proof! That is to say, you will need to prove that P2 can always win under the aforementioned condition, but whenever that condition is not met, you have to prove that it is now P1 that can always win!

ansver
Answers: 2

Other questions on the subject: Mathematics

image
Mathematics, 21.06.2019 17:00, JvGaming2001
Scarlet bought three pairs of sunglasses and two shirts for $81 and paula bought one pair of sunglasses and five shirts for $105 what is the cost of one pair of sunglasses and one shirt?
Answers: 2
image
Mathematics, 21.06.2019 17:30, lorihab1970
Can someone me and plz do the problem
Answers: 2
image
Mathematics, 21.06.2019 22:00, danielahalesp87vj0
18 16 11 45 33 11 33 14 18 11 what is the mode for this data set
Answers: 2
image
Mathematics, 21.06.2019 22:40, alialoydd11
Afunction g(x) has x-intercepts at (, 0) and (6, 0). which could be g(x)? g(x) = 2(x + 1)(x + 6) g(x) = (x – 6)(2x – 1) g(x) = 2(x – 2)(x – 6) g(x) = (x + 6)(x + 2)
Answers: 1
You know the right answer?
Using strong induction (so, neither weak nor "weak++") on the number of matches in the pile, prove t...

Questions in other subjects:

Konu
English, 17.09.2020 14:01
Konu
Health, 17.09.2020 14:01
Konu
Mathematics, 17.09.2020 14:01
Konu
Mathematics, 17.09.2020 14:01
Konu
Mathematics, 17.09.2020 14:01
Konu
Mathematics, 17.09.2020 14:01
Konu
Mathematics, 17.09.2020 14:01
Konu
Biology, 17.09.2020 14:01
Konu
English, 17.09.2020 14:01
Konu
Mathematics, 17.09.2020 14:01